LSAT and Law School Admissions Forum

Get expert LSAT preparation and law school admissions advice from PowerScore Test Preparation.

 Administrator
PowerScore Staff
  • PowerScore Staff
  • Posts: 8919
  • Joined: Feb 02, 2011
|
#33986
Please post your questions below!
 karen4300
  • Posts: 9
  • Joined: Jun 28, 2018
|
#47377
I chose D). I didn't really understand the question stem. Could someone help me understand why E is right?
 nmgee
  • Posts: 15
  • Joined: Jun 01, 2018
|
#47481
Hi. I chose E because I thought it was just slightly better than C. I thought both answers seemed strong.

Are we supposed to not consider this ad hominem solely because the view is being criticized not explicitly the person/people holding the view? How should we distinguish between an argument attacking the view vs. the viewholder? I think this would better help me rule out C, because I thought the "some say X and this dangerous point of view...would have stifled research" implied the people holding the view were also being argued against in addition to he view itself. Thanks in advance for your help!
User avatar
 Jonathan Evans
PowerScore Staff
  • PowerScore Staff
  • Posts: 726
  • Joined: Jun 09, 2016
|
#47503
Hi, NMGee,

Looks like you've mostly got it. The argument does not attack directly the people who are against the megatelescope. It attacks their view. The argument states the view is "dangerous." We cannot assume that this assertion is an ad hominem/source argument directed against the opponents directly.

Thus, the following statement is incorrect:
I thought the "some say X and this dangerous point of view...would have stifled research" implied the people holding the view were also being argued against in addition to he view itself.
The above statement is directed against the view, not its proponents.

Does this make sense? Good job.
 LSAT2020
  • Posts: 31
  • Joined: Jun 24, 2020
|
#76749
Could we say that this argument is weak due to the analogy that the observatory director is attempting to use as support for the argument? The director is comparing the applications of Newton, Einstein, and Maxwell's to that of the astronomers, right? The observatory director seems to be assuming that the work that the astronomers will do with the telescope is going to be as groundbreaking to science as that of Newton, Einstein, and Maxwell's. I feel like the analogy could've worked if the director had given us proof that the work by the astronomers is as ground-breaking as that of Newton, Einstein, and Maxwell.
 Jeremy Press
PowerScore Staff
  • PowerScore Staff
  • Posts: 1000
  • Joined: Jun 12, 2017
|
#76812
Hi LSAT2020,

Yes, that's exactly right, and you're also right about how the argument could've been strengthened. Also notice that, since you've identified the flaw as being analogy-based, it's not at all surprising that the correct answer (E) refers to a problematic "comparison."

Nice work!

Jeremy

Get the most out of your LSAT Prep Plus subscription.

Analyze and track your performance with our Testing and Analytics Package.